Show Noether Currents Given Dirac Eq & Conjugate

In summary, to show that j^\mu = \bar{\psi} \gamma^\mu \psi is a Noether current, we can use the formula for Noether currents from scalar field theory and adapt it to fermions. Given the symmetry operation of rotating the phase of the spinor, we can use this in the formula to show that j^\mu is indeed the Noether current. To do this, we first write down the classical Lagrangian for the Dirac field, which is invariant under this phase rotation symmetry. Then, using the formula for Noether currents, we substitute in the appropriate values and find that j^\mu = \bar{\psi} \gamma^\mu \psi, confirming that it is indeed a No
  • #1
latentcorpse
1,444
0
Given the Dirac equation and its conjugate, how do I show that [itex]j^\mu = \bar{\psi} \gamma^\mu \psi[/itex] is a Noether current?

Is there even a standard formula for Noether currents or does it vary depending on each individual case? And if it does vary, how do I go about figuring how to apply it to each particular example?

Thanks!
 
Physics news on Phys.org
  • #2
The Noether current is always associated with a symmetry. Given the symmetry operation and Lagrangian, you can always compute the Noether current. The general formula for Noether currents from scalar field theory can be adapted to fermions.

Given the Noether current, it's a little bit trickier to invert the relationship, but you can usually do it on a case by case basis.
 
  • #3
fzero said:
The Noether current is always associated with a symmetry. Given the symmetry operation and Lagrangian, you can always compute the Noether current. The general formula for Noether currents from scalar field theory can be adapted to fermions.

Given the Noether current, it's a little bit trickier to invert the relationship, but you can usually do it on a case by case basis.

Sorry. I still don't understand what I'm supposed to do? You talk about inverting the relationship - am I expected to find the symmetry, and if so, how?

Thanks.
 
  • #4
There is a formula to get the Noether current corresponding to a symmetry of a given Lagrangian. You can find a derivation of this formula for classical field theories embedded in http://www.physics.utoronto.ca/~luke/PHY2403/References.html" - start at page 40. I think it is for a scalar theory but you can figure out how to adapt it to a field with multiple components.

To apply this I think you will need to first figure out or guess what symmetry the current you are given corresponds to.
 
Last edited by a moderator:
  • #5
The_Duck said:
There is a formula to get the Noether current corresponding to a symmetry of a given Lagrangian. You can find a derivation of this formula for classical field theories embedded in http://www.physics.utoronto.ca/~luke/PHY2403/References.html" - start at page 40. I think it is for a scalar theory but you can figure out how to adapt it to a field with multiple components.

To apply this I think you will need to first figure out or guess what symmetry the current you are given corresponds to.

"guesswork" doesn't seem very mathematically precise though?

Also, what is [itex]\Pi[/itex] in the notation in your notes?

Thanks.
 
Last edited by a moderator:
  • #6
latentcorpse said:
"guesswork" doesn't seem very mathematically precise though?

Also, what is [itex]\Pi[/itex] in the notation in your notes?

Thanks.

Rereading a little I see the notes do indeed seem to be treating the case of a field [itex]\phi_a[/itex] with several components indexed by 'a'.

[itex]\Pi_a^\mu = \frac{\partial \mathcal{L}}{\partial (\partial_\mu \phi_a)}[/itex] is the "canonically conjugate momentum" to the field component [itex]\phi_a[/itex]:

The solution process I am imagining here is: write down the classical Lagrangian which gives an equation of motion that is the Dirac equation (you can look this up). Find the symmetries of this Lagrangian and plug them into the formula for Noether currents, until you find the one that gives the desired current.

I only learned this stuff very recently, so perhaps you are supposed to be doing something else here. Have you done classical field theory from Lagrangians? You can read about it in those notes if not, if that's what you're supposed to be doing.
 
  • #7
The_Duck said:
Rereading a little I see the notes do indeed seem to be treating the case of a field [itex]\phi_a[/itex] with several components indexed by 'a'.

[itex]\Pi_a^\mu = \frac{\partial \mathcal{L}}{\partial (\partial_\mu \phi_a)}[/itex] is the "canonically conjugate momentum" to the field component [itex]\phi_a[/itex]:

The solution process I am imagining here is: write down the classical Lagrangian which gives an equation of motion that is the Dirac equation (you can look this up). Find the symmetries of this Lagrangian and plug them into the formula for Noether currents, until you find the one that gives the desired current.

I only learned this stuff very recently, so perhaps you are supposed to be doing something else here. Have you done classical field theory from Lagrangians? You can read about it in those notes if not, if that's what you're supposed to be doing.

okay so the Lagrangian for the Dirac field is

[itex]L= \bar{\psi(x)} \left( i \gamma^\mu \partial_\mu - m \right) \psi(x)[/itex]

Now in a set of notes that we have, I have read that the Dirac Lagrangian is invariant under rotating the phase of the spinor, [itex]\psi \rightarrow e^{-i \alpha} \psi[/itex], and that it is this symmetry that gives rise to this particular Noether current.

So I guess the next step is to use this symmetry in the formula to show this is the Noether current.

On page 43 of your notes, the formula for the Noether current is:

[itex]j^\mu = \Pi^\mu_a D \phi_a - F^\mu[/itex]

Is this the general form that we can adapt to any situation?

If so, I can sub [itex]\psi[/itex] for [itex]\phi_a[/itex] to get

[itex]j^\mu = \frac{ \partial L}{ \partial ( \partial_\mu \psi) } D \psi - F^\mu[/itex]

Is this ok so far?
 
Last edited:
  • #8
Yeah, so now you will need to calculate that expression for the phase-rotation symmetry.
 
  • #9
The_Duck said:
Yeah, so now you will need to calculate that expression for the phase-rotation symmetry.

well in your notes it says [itex]D \psi = \frac{ \partial \psi}{ \partial \lambda} |_{ \lambda = 0 }[/itex] but I don't understand how to evaluate this...

actually, i have an idea...
if we have a transformation [itex]\phi_a(x) = \phi_a(x,0) \rightarrow \phi_a(x, \lambda)[/itex]
then[itex]D \phi_a = \frac{\partial \phi_a}{\partial \lambda} |_{\lambda=0}[/itex]

and so in our case [itex]\psi(x) \rightarrow e^{-i \alpha} \psi(x) = \psi(x, \alpha)[/itex]
and so [itex] D \psi = \frac{\partial \psi}{\partial \alpha} |_{\alpha=0}=-i \psi[/itex]

and then since [itex]\frac{\partial L}{\partial ( \partial_u \psi)} = i \bar{\psi} \gamma^\mu [/itex] we find

[itex]j^\mu = i \bar{\psi} \gamma^\mu ( -i \psi) = \bar{\psi} \gamma^\mu \psi[/itex] as required.

The only slight "hiccup" with this is that [itex]D \phi_a[/itex] would have to be defined as [itex]D \phi_a = \frac{\partial \phi_a(x, \lambda)}{\partial \lambda}|_{\lambda=0}[/itex] whereas in your notes it is just written [itex]D \phi_a = \frac{\partial \phi_a}{\partial \lambda} |_{\lambda=0}[/itex]. Is this alright?

And also, how do we justify setting [itex]F^\mu=0[/itex]
 
Last edited:
  • #10
latentcorpse said:
\The only slight "hiccup" with this is that [itex]D \phi_a[/itex] would have to be defined as [itex]D \phi_a = \frac{\partial \phi_a(x, \lambda)}{\partial \lambda}|_{\lambda=0}[/itex] whereas in your notes it is just written [itex]D \phi_a = \frac{\partial \phi_a}{\partial \lambda} |_{\lambda=0}[/itex]. Is this alright?

Yes, the arguments of the phi are just left unwritten in that equation.

latentcorpse said:
And also, how do we justify setting [itex]F^\mu=0[/itex]

Here [itex]F^\mu[/itex] is defined by

[itex]\frac{\partial \mathcal{L}}{\partial \lambda}|_{\lambda = 0} = \partial_\mu F^\mu[/itex]

so you can figure out what F must be based on how the Lagrangian changes under your symmetry. This is how you can tell if you have a symmetry and thus a conserved current: if you can write the left hand side of the above in the form of the right hand side, then the notes show how you can use this information to construct a conserved current. In this problem, F is very simple because the Lagrangian is completely unchanged by the symmetry transformation, so the left hand side is zero.

Have you worked through the derivation given in those notes? I think if you do you'll see why this recipe for getting a conserved current works.
 
  • #11
The_Duck said:
Yes, the arguments of the phi are just left unwritten in that equation.



Here [itex]F^\mu[/itex] is defined by

[itex]\frac{\partial \mathcal{L}}{\partial \lambda}|_{\lambda = 0} = \partial_\mu F^\mu[/itex]

so you can figure out what F must be based on how the Lagrangian changes under your symmetry. This is how you can tell if you have a symmetry and thus a conserved current: if you can write the left hand side of the above in the form of the right hand side, then the notes show how you can use this information to construct a conserved current. In this problem, F is very simple because the Lagrangian is completely unchanged by the symmetry transformation, so the left hand side is zero.

Have you worked through the derivation given in those notes? I think if you do you'll see why this recipe for getting a conserved current works.

Is this formula for [itex]F^\mu[/itex] true for all examples?

Also, surely if the LHS is 0, [itex]F^\mu[/itex] doesn't ahve to be 0, it just has to be constant, no?
 
  • #12
latentcorpse said:
Is this formula for [itex]F^\mu[/itex] true for all examples?

If F satisfies that equation then the other formula that uses F gives a conserved current. Work through the derivation to see why F is defined the way it is.

latentcorpse said:
Also, surely if the LHS is 0, [itex]F^\mu[/itex] doesn't ahve to be 0, it just has to be constant, no?

Indeed. And if you add a constant 4-vector to a conserved current you still get a conserved current.
 
  • #13
The_Duck said:
If F satisfies that equation then the other formula that uses F gives a conserved current. Work through the derivation to see why F is defined the way it is.



Indeed. And if you add a constant 4-vector to a conserved current you still get a conserved current.

so our answer for the Noether current is defined up to a constant and then what - do we argue that we might as well just set this constant equal to zero?
 
  • #14
The boundary conditions at infinity would <kill> any constant added to the 4-current.
 

Related to Show Noether Currents Given Dirac Eq & Conjugate

1. What are Noether currents in relation to the Dirac equation?

Noether currents are quantities that are conserved in a specific physical system. In the context of the Dirac equation, these currents represent symmetries in the system, such as conservation of energy and momentum.

2. How are Noether currents related to the conjugate of the Dirac equation?

The conjugate of the Dirac equation is a mathematical representation of the complex conjugate of the equation. This allows for the calculation of Noether currents, as they are defined by certain mathematical operations on the conjugate of the Dirac equation.

3. Can you give an example of calculating Noether currents using the Dirac equation and its conjugate?

One example is the calculation of the electromagnetic current in quantum electrodynamics. By applying the Noether theorem to the Dirac equation and its conjugate, it is possible to derive the conserved current that describes the interaction between charged particles and electromagnetic fields.

4. What is the significance of Noether currents in physics?

Noether currents play a crucial role in understanding the symmetries and conservation laws in physical systems. They provide a powerful tool for understanding the underlying principles and behavior of particles and fields in the universe.

5. Are there any other applications of Noether currents besides in the Dirac equation?

Yes, Noether currents have been applied in various areas of physics, including quantum field theory, general relativity, and statistical mechanics. They have also been used in the study of fluid dynamics and other complex systems.

Similar threads

  • Advanced Physics Homework Help
Replies
1
Views
803
  • Advanced Physics Homework Help
Replies
0
Views
83
Replies
3
Views
1K
  • Advanced Physics Homework Help
3
Replies
95
Views
5K
  • Advanced Physics Homework Help
Replies
8
Views
3K
  • Advanced Physics Homework Help
Replies
1
Views
965
  • Advanced Physics Homework Help
Replies
2
Views
989
  • Advanced Physics Homework Help
Replies
4
Views
2K
  • Advanced Physics Homework Help
Replies
2
Views
3K
  • Advanced Physics Homework Help
Replies
30
Views
5K
Back
Top